1. quadrilateral with 4 right angles

Answers

Answer 1

Answer:

Square

Step-by-step explanation:


Related Questions

what is the area of the figure?

Answers

Answer:

182 m

Step-by-step explanation:

The shape is a trapezoid. The area of the trapezoid formula is [tex]\frac{a + b}{2}h[/tex].

The two bases (AKA a and b) are 8 + 18.

The height is 14.

[tex]\frac{18+8}{2} 14[/tex], Add 18 + 8

[tex]\frac{26}{2}14[/tex], Divide 26 by 2

[tex]13 *14[/tex], 182

When a constant force acts
upon an object, the acceleration of the object varies Inversely with Its mass. When a certain constant force acts upon an object witt
mass 5 kg, the acceleration of the object Is 15 m/s-
If the same force acts upon another object whose mass is 3 kg, what is this object's acceleration?

Answers

Answer:

25

Step-by-step explanation:

First expression: A= k/M

when A = 15 and M = 5, k = A×M = 75

therefore, when M = 4, A = 75/3 = 25

Find the quotient of 36 and -4.

Answers

The quotient of 36 and -4 is -9.

36/-4 = -9

4. triangle FBT is similar to triangle ADQ with a scale factor of 5/3. Complete the table below.

Sides Angles

BF = 15 B = 85

BT = F = 40

FT = 21 T =

AD = 9 A =

DQ = 6 D =

AQ = Q = 55

Answers

Triangle FBT is similar to triangle ADQ with a scale factor of 5/3. AD = 9; BF = 15, DQ = 6; BT = 10, FT = 21; AQ = 12.6

∠F = ∠A = 40°, ∠Q = ∠T = 55°, ∠B = ∠D = 85°

What are similar triangles?

Similar triangles are triangles that have the same shape and the ratio of their corresponding sides are in the same proportion. Corresponding angles are equal.

triangle FBT is similar to triangle ADQ with a scale factor of 5/3.

AD = 9; BF = 9 * 5/3 = 15

DQ = 6; BT = 6 * 5/3 = 10

FT = 21; AQ = 21 * 3/5 = 12.6

∠F = ∠A = 40°, ∠Q = ∠T = 55°, ∠B = ∠D = 85°

Find out more on similar triangles at: https://brainly.com/question/2644832

If Mrs.davis serves each guest 1/6 of a pie what is the greatest number of guests she can serve

Answers

Well if the whole pie is divided into 1/6ths it would only be 6 people before the entire pie ran out
Mrs. Davis can serve a maximum of 6 guests if she gives 1/6 of the pie to each guest.

Help help help help help

Answers

Answer:

A' = (- 3, 0)

Step-by-step explanation:

[tex]A' = (x + 3, \: y - 2) \\ \\ \implies \: A' = ( - 6+ 3, \: 2 - 2) \\ \\ \implies \: A' = ( - 3, \: 0) [/tex]

Answer:

A' = (-3, 0)

Step-by-step explanation:

Pre-Image coordinate:

A (-6, 2)

Translation: (x, y) --> (x + 3, y - 2)

(-6 + 3, 2 - 2)

(-3 0)

Image coordinate:

A' = (-3, 0)

hope this helps!! p.s. i really need brainliest :)

Show that f(x) = 3x
4 − 2x
2 + 5 is an even function.

Answers

Answer:
Step-by-step explanation:
This f(x) is an even function.
Explanation:
An even function is one where f(−x)=f(x) for all x in the domain.
An odd function is one where f(−x)=−f(x) for all x in the domain.
In the case of f(x)=3x4−x2+2 we find:
f(−x)=3(−x)4−(−x)2+2=3x4−x2+2=f(x)
So f(x) is an even function.
Actually, there is a shortcut with polynomial functions:
If all of the terms have even degree then the function is even.
If all of the terms have odd degree then the function is odd.
Otherwise the function is neither odd nor even.
in our example, 3x4 has degree 4, −x2 has degree 2 and 2 has degree 0. So all of the terms are of even degree and the function is even

Find the value of y and z

Answers

[tex]\qquad\qquad\huge\underline{{\sf Answer}} \huge♨[/tex]

In the given figure, NQ acts as a diameter and since diameter subtends 90° at the arc of the circle, so we can conclude that ~

[tex]\qquad \sf  \dashrightarrow \:3y = 90 \degree[/tex]

[tex]\qquad \sf  \dashrightarrow \:y = 90 \degree \div 3[/tex]

[tex]\qquad \sf  \dashrightarrow \:y = 30 \degree [/tex]

Now, Let's use Angle sum property of Triangle to solve for z :

[tex]\qquad \sf  \dashrightarrow \:4z + 3y + 50 \degree = 180 \degree[/tex]

[tex]\qquad \sf  \dashrightarrow \:4z +90 \degree + 50 \degree = 180 \degree[/tex]

[tex]\qquad \sf  \dashrightarrow \:4z = 180 \degree - (50 \degree + 90\degree)[/tex]

[tex]\qquad \sf  \dashrightarrow \:4z = 180 \degree - (140\degree)[/tex]

[tex]\qquad \sf  \dashrightarrow \:4z = 40 \degree[/tex]

[tex]\qquad \sf  \dashrightarrow \:z = 40 \degree \div 4[/tex]

[tex]\qquad \sf  \dashrightarrow \:z = 10 \degree [/tex]

I hope you understood the procedure, you can ask me in comments if you have any doubts.

As you move to the
on a number line the values of the numbers
Which choice best completes the sentence?
right; increase
right; decrease
left; increase
left; remain the same

Help please I’m stuck on this question!

Answers

Answer:

Based on the argument behind the correctness of the assumption is impossible, so we rule that the assumption is correct, now is the answer

The second answer from above

Find three points that lie on the circle

Answers

Answer:

Use (a-b)2 formula that is a2-2ab+B2

I think it will help you

Suposse X=a

y= B

And then use formula

In base 2 (Binary system), which equation is false?
*(explain answer in detail)

a) 11+1=101
b) 1x10=10
c) 11x11=1001
d) 100/1=100
e) 1+1+1=11

Answers

Answer:

it is a

Step-by-step explanation:

because 11 +1 8n binary is qlways 100 bbecause 11 is in the 4th in the truth table of binary numbers and 100 is in the 5th so 11 + 1 = 4+1

100 = 5

pls mark it as the brainlyest answer

Answer:

It's a.

Step-by-step explanation:

11 binary = 2 + 1 = 3 in decimal.

So 11 + 1 = 3 + 1 = 4 in decimal.

and 4 in binary = 100  (2^2 + 0 + 0)

What number is one-eighth of three hundred and sixty?

Answers

Answer:

45

Step-by-step explanation:

Have a great day!

#EquestrianLion

Trigonometric Equations

Solve the equation over the interval [0°, 360°)

cos(theta) = -1/2

Answers

Answer:

[tex]\frac{2\pi }{3};\frac{4\pi }{3}.[/tex]

Step-by-step explanation:

1) cos(θ)=-0.5;

theta=±2π/3+2πn, where n∈Z;

2) if θ∈[0;360], then

[tex]\theta=\frac{2\pi }{3}; \frac{4\pi }{3}.[/tex]

Suppose we want to choose 7 letters, without replacement, from 12 distinct letters. (a) How many ways can this be done, if the order of the choices is not relevant?

Answers

Answer:

3991680 or 792

Step-by-step explanation:

12P7 = 3991680

or

12C7 = 792

P = permutation

C = combination

Sorry, I am not sure which answer is correct or if any of the answers are correct.

which of the following steps were applied to ABCD to obtain A'B'C'D'?

A. shifted 3 units left and 2 units up
B. shifted 2 units left and 4 units up
C. shifted 2 units left and 3 units up
D. shifted 3 units left and 4 units up ​

Answers

Answer:

A. shifted 3 units left and 2 units up

Answer:

shifted 3 units left and 2 units up

100 POINTS! ! !
Exploiting will be reported.


Chase deposits $6,604.80 into a savings account that pays a simple annual interest rate of 3.22%. How much interest will he earn after 4 years?

SHOW WORK and will give brainlist.

Answers

Answer:

$850.69824

Step-by-step explanation:

simple interest formula : (principal * rate of interest * time)/100

(6,604.80 * 3.22 * 4)/100

$850.69824

Answer:

$850.70 (nearest cent)

Step-by-step explanation:

Simple interest formula:   A = P(1 + rt)

where:

A = final amountP = principal (initial balance)r = annual interest rate (in decimal form)t = time (in years)

Given:

P = $6,604.80r = 3.22% = 0.0322t = 4 years

Substituting given values into the formula, and solving to find A:

⇒ A = 6604.8(1 + 0.0322 · 4)

       = 6604.8(1.1288)

       = $7455.50 (nearest cent)

Therefore, to find the amount of interest earned, subtract the principal from the final amount:

⇒ I = A - P

     = 7455.50 - 6604.80

     = $850.70 (nearest cent)

what is the slope of 5x-4y=3

Answers

Answer:

5/4

Step-by-step explanation:

Hi there!

We are given the line 5x-4y=3, and we want to find the slope of it.

The line is currently in standard form (ax+by=c); one way to find the slope of the line is to convert it into slope-intercept form (y=mx+b)

To do that, we need to isolate y on one side (in other words, we need to solve the equation for y)

First, subtract 5x from both sides

5x-4y=3

-5x     -5x

____________

-4y = -5x + 3

Divide both sides by -4

[tex]y=\frac{5}{4} x - \frac{3}{4}[/tex]

In slope intercept form, m represents the slope; in this problem, 5/4 takes the place of where m should be, meaning 5/4 is the slope.

Hope this helps!

See more on this topic here: https://brainly.com/question/18929849

The grandma of a 6-year-old is asking them 2x3 what is 2x3

Answers

Answer:

The answer of 2 x 3 would be 6

Step-by-step explanation:

The reason why 2 x 3 or 2 times 3 would be six/6. Is because if you had 2 pairs of 3 items, you would get a total of 6 items. And if you had 3 pairs of 2 items, you would still have a total of 6 items.

Use Demos Graphing Calculator
Rob works part time at the Fallbrook Riding Stable. He makes $5 an hour exercising
horses and $10 an hour cleaning stalls. Because Rob is a full-time student, he can
work no more than 12 hours per week. However, he must make at least $60 per
week.

Which of the following is a possible solution for this system of inequalities?

A. 1 hour exercising and 1 hour cleaning

B. Two hours exercising and five hours of cleaning

C. Seven hours of exercising and eight hours of cleaning

D. Two hours exercising and eight hours cleaning

Answers

Answer:

D. Two hours exercising and eight hours cleaning

Step-by-step explanation:

Rob works part time at the Fallbrook Riding Stable. He makes $5 an hour exercising horses and $10 an hour cleaning stalls. Because Rob is a full-time student, he can work no more than 12 hours per week. However, he must make at least $60 per week.

x = hrs. exercising horses

y = hrs. cleaning stalls

Equations:

Hours he can work per week:

x + y ≤ 12

Amount of money he can make per week:

5x + 10y ≥ 60

Now, graph your equations into desmos.

As you can see in the graph, the correct answer is D (2, 8)

Check your answer by hand:

x + y ≤ 12

2 + 8 ≤ 12

10 ≤ 12

This statement is correct

5x + 10y ≥ 60

5(2) + 10(8) ≥ 60

10 + 80 ≥ 60

90 ≥ 60

This statement is correct

Therefore, the correct answer is D

Hope this helps!

z varies jointly with x and y. If x = 2, y = 3, and z = 4, write the variation equation and find z when x = −6 and y = 2.

Answers

Step-by-step explanation:

that means

z = k×x×y

4 = k×2×3 = k×6

k = 4/6 = 2/3

so, when x = -6 and y = 2, we get

z = 2/3 × -6 × 2 = 2 × -2 × 2 = -8

Please help
I don’t know what to do with this

Answers

Answer:

The prediction should be 5 via interpolation.

Step-by-step explanation:

Based on the values you have, the linear equation is f(x)=x-1. So replace x with the number you're looking for (6).

In a general sense, to extrapolate is to infer something that is not explicitly stated from existing information. Interpolation is an estimation of a value within two known values in a sequence of values.

Can yall help me with the answers?

Answers

Answer:

Number 3 is B

Step-by-step explanation:

B is number 3

Answer:

Lakaka 1 by Lakaka 1 vx wfs

Aaron has $25 to spend at the carnival. Admission is $4.00 and the ride tickets are $1.25 each. What is the maximum number of ride tickets that Aaron can buy? Write and solve the inequality.

Answers

16 rides aaron can ride

Answer:

25 dollars divided by 1.25 dollars gives a total of 20 tickets Aaron can buy

Can you help me? PLEASE

Answers

Hello!

Let's simplify the expression

6-2x+5+4x

Combine like terms:

6+5-2x+4x

11+2x

And that's the final answer, because 11 and 2x are not like terms (and that's Tyler's error. He shouldn't have combined 11 and 2x.

This is the correct way to simplify:

6+5-2x+4x

11+2x

Hope everything is clear.

Let me know if you have any questions!

#NeverGiveUp

:-)

Help please if ur good at math

Answers

Part (a)

  [tex]\int\limits^6_0 {f(x)} \, dx =\int\limits^3_0 {f(x)} \, dx +\int\limits^6_3 {f(x)} \, dx =8+(-7) = 1[/tex]

Part (b)

 [tex]\int\limits^3_6 {f(x)} \, dx =-\int\limits^6_3 {f(x)} \, dx =-(-7) = 7\\[/tex]

Part (c)

[tex]\int\limits^3_3 {f(x)} \, dx =0[/tex]

Part (d)

[tex]\int\limits^6_3 {-5f(x)} \, dx =-5* \int\limits^6_3 {f(x)} \, dx=-5 * -7 =35[/tex]

Hope that helps!

Simplify the expression. Enter the answer in the box. 5 1/6 - 7 1/3

Answers

Answer:

-15.6666667

Step-by-step explanation:

first of all divide both side then minus the divide number

2. Evaluate the function rule for the given value. y = 4.2 for x = -6​

Answers

Answer:
y= 4 x 2(6)
y= 4 x -12
y= -48
Step-by-step explanation:
y= 4 x 2(6)
y= 4 x -12
y= -48

Please answer the following questions

An absolute value inequality can be solved using a union or an intersection depending on which sign it is true or false

Answers

Answer:

True????????????????????

The statement that "An absolute value inequality can be solved using a union or an intersection depending on which sign it is" is true.

What is Absolute Value Inequality?

Absolute value inequalities are kind of inequality expressions which includes the absolute value symbol in the expression.

For example : |x - 5| < 4 is an absolute value inequality.

Given statement is that,

An absolute value inequality can be solved using a union or an intersection depending on which sign it is.

If an absolute value inequality has a sign as "> 0", then the solution contains every real numbers but not values which makes the inequality 0.

in this case, union is used.

Now, if the inequality is "<" or "≤" with a positive number, the we use the intersection.

Hence the given statement is true.

Learn more about Absolute value Inequalities here :3

https://brainly.com/question/30201926

#SPJ2

If 12%of X is 306, what is X equivalent to?

Answers

Answer:

X = 2550

Step-by-step explanation:

Convert 12% to a decimal:

⇒ 12% = 12/100 = 0.12

   12% of X is 306

⇒ 0.12X = 306

⇒ X = 306 ÷ 0.12

⇒ X = 2550

Answer:

2550

Step-by-step explanation:

Given expression: 12% of x is 306

We can make the following points of the expression:

    1. The word "of" means multiplication (×).

⇒ 12% × x is 306

    2. The percent is always divided by 100 in fraction.

⇒ 12/100 × x is 306

    3. The word "is" means equal or equivalent.

⇒ 12/100 × x = 306

Now, let's simplify the expression.

⇒ 12x/100 = 306

⇒ 12x = 306 × 100

⇒ 12x/12 = 306 × 100/12

x = 51 × 50 = 2550

How large a sample must a pollster take in order to estimate with 95% confidence and to within 3 percentage points, the proportion of voters who are in favor of a certain measure

Answers

Answer:   1068

========================================================

Work Shown:

[tex]n = \hat{p}*(1-\hat{p})\left(\frac{z}{E}\right)^2\\\\n \approx 0.5*(1-0.5)\left(\frac{1.96}{0.03}\right)^2\\\\n \approx 1067.111\\\\n \approx \boldsymbol{1068}\\\\[/tex]

Notes:

At 95% confidence, the z critical value is roughly z = 1.96 which is determined using a Z table.E = 0.03 to represent the 3% error. We're not told the value of [tex]\hat{p}[/tex], so we assume the most conservative estimate of 0.5Always round up to the nearest integer. The value 1067.111 is closer to 1067, but we round up to 1068 to clear the hurdle needed.
Other Questions
Sara and her friends are enjoying a bag of candy-coated chocolates. Just for fun, they remove 80 pieces of candy from the bag to check their color. The outcomes are shown in the table.Candy Color Number of Candiesbrown or orange 25green or yellow 28red or blue 27Which statement is true based on the results in the table?A.All of the outcomes are equally likely because their relative frequencies are similar.B.All of the outcomes are equally likely because the relative frequency of picking brown or orange candies is greater than that of picking green or yellow.C.All of the outcomes are not equally likely because there is a lot of variation in their relative frequencies.D.All of the outcomes are not equally likely because the relative frequency of picking green or yellow candies is the highest.E.All of the outcomes are not equally likely because the relative frequency of picking red or blue candies is less than that of picking green or yellow. hello, anyhelp? if anybody likes they can answer, show work write an expresstion that is equivalent to 1/3x+3/4+2/3x-1/4-2/3x A village loses 20% of their goats to the flood and 5% die from diseases. There are 8084 goats left, how many goats were their originally. How do the names of molecular compounds differ from the names of ionic compounds. Find the volume of a right circular cone that has a height of 13.1 in and a base with a diameter of 5.7 in Nuclear-generated power plants are heavily subsidized. Do you think the market price of these plants should include the costs of the nuclear fuel cycle? Explain. Csar chvez championed the economic rights of which group?. Find the length of PF Write each expression without the absolute value symbol |3x+12| Which type of rhythm is demonstrated in the stairs? A Flowing B Progressive C Random D Regular Hydrogen peroxide and potassium permanganate balanced equation A fan base is, in most ways, quite similar to a customer base.A. FalseB. True A student wants to determine the effect of soil type on plant growth. He set up 3 pots as shown below. How could this experiment be improved?A - There is nothing wrong with this experiment.B - The three pots should have different amounts of waterC - All three pots should have the same kind of seed, the same amount of light, and the same amount of water.D - All three pots should have the same kind of soil. urgent help would be appreciated 1980s why did unions decline? Luke bought a video game system that costs $99.95 and games that cost $19.99 each. Bella bought a game system that costs $125.79 and games that cost $15.99 each. Luke spent no more than what Bella spent.Which inequality represents this situation?99.95 + 19.99x < 125.79 + 15.99x99.95 + 19.99x < 125.79 + 15.99x99.95 + 19.99x > 125.79 + 15.99x99.95 + 19.99x > 125.79 + 15.99x 24/k=4what is kshow your steps A 1000 kg rocket carrying 25 kg of fuel and oxygen rises at a velocity of 305 m/s. If all the mass of fuel and oxygen is burned to form gases of combustion, what is the downward velocity of these gases? Schaad Corporation has entered into an eight-year lease for a piece of equipment. The annual payment under the lease will be $2,500, with payments being made at the beginning of each year. If the discount rate is 14%, whats the present value of the lease payments? Ignore income taxes in this problem